Assumption

Practice Questions

LSAT Logical Reasoning › Assumption

Page 1 of 5
10 of 49
1

A law degree is necessary to practice as an attorney. Further, no one who has demonstrated moral turpitude may be admitted to practice law. Consequently, Smith, who was convicted of embezzlement several years ago, cannot be admitted to practice law.

The argument follows logically if which of the following is assumed?

2

A medical degree is necessary for appointment to the hospital's board of directors. Further, no one having more than a five-percent equity stake in a pharmaceutical company can be appointed to the board of directors. Consequently, Dell, a practicing physician with a PhD in bioethics, cannot be appointed the hospital's treasurer, since he owns fifteen percent of PillCo, a pharmaceutical company.

The argument’s conclusion follows logically if which one of the following is assumed?

3

Live theater has been in competition with more modern media options for years. Today, there are a vast number of entertainment options and there is easy access to high quality entertainment technology, such as sophisticated home theater systems. Fewer people watch live theater than in the past. As a result, live theater has lost the competition with modern media.

Which of the following, if true, would allow the conclusion to be logically drawn?

4

It is wrong to condemn the eating of lobster. Lobsters do not have the kind of cognitive abilities that permits meaningful self-awareness, and that sort of cognitive ability is essential to experience suffering.

The conclusion above follows logically if which one of the following is assumed?

5

Among the various models of racing cars used in top-level racing competitions, one cannot predict a car's ability to reach maximum speed simply by the horsepower of the motor. The efficiency of a motor's fuel-injection system varies significantly, even between racing cars with motors of comparable horsepower.

The argument's conclusion is properly drawn if which one of the following is assumed?

6

Standard sheets of printer paper do not vary in the amount of wood pulp that they contain. Twenty-five percent of the wood pulp contained in a certain class of sheets of standard printer paper (Class B) was recycled from used sheets of standard printer paper of a different class (Class A). Since all Class A sheets were recycled into Class B sheets and since the amount of material other than wood pulp in a sheet of standard printer paper is negligible, it follows that Class B contains 4 times as many sheets of paper as Class A.

The conclusion of the argument follows logically if which one of the following is assumed?

7

The government of a country mandates that all people who are citizens of that country vote in a general election and only people who are citizens shall vote. Therefore, no people who have immigrated to the country six months or less prior to the general election will vote in the general election, but some immigrants will vote in the general election.

The conclusion above follows logically if which of the following answers is assumed.

8

Scientists are now reporting that recent studies show the drug has been extremely effective in masking the harmful side effects of the disease. This breakthrough gives hope to many patients that scientists are very close to finding an appropriate medicine to fully cure the disease.

Which of the following statements is a sufficient assumption for the argument in the above passage?

9

It is important that each driver have an insurance policy before driving on the road. Otherwise, people may not be compensated when they are injured in car accidents. Everyone injured in accidents deserves to be compensated.

The argument assumes which of the following?

10

Soccer players are faster than baseball players because soccer players use more strenuous training programs. Baseball players should use the same training programs as soccer players to become more athletic. More athletic players will be more successful.

Which of the following, if assumed, would allow the conclusion above to be properly drawn?

Page 1 of 5
Return to subject